LSAT and Law School Admissions Forum

Get expert LSAT preparation and law school admissions advice from PowerScore Test Preparation.

 Administrator
PowerScore Staff
  • PowerScore Staff
  • Posts: 8916
  • Joined: Feb 02, 2011
|
#40921
Complete Question Explanation
(The complete setup for this game can be found here: lsat/viewtopic.php?t=1007)

The correct answer choice is (C)

The question stem requires you to add a course to R that would fully determine all four of Alicia’s courses. With R selected, the current choices appear as follows:
PT59_Game_#3_#13_diagram 1.png
The course that is selected for this question must determine which of G or W is selected, and that requirement limits the pool of likely candidates to P and S9, because S9 affects W, and P affects S9 (S3 is a random, and has no effect when selected, other than to occupy space). As S9 is not among the choices, the likely answer, then, is P. From the inferences you can see what occurs when P is selected, but if not, simply apply the rules. Let us examine what occurs when P is selected:
  • When P is selected, from the fourth rule S9 is selected. When S9 (and P) is selected, then from the third rule W cannot be selected, which results in G being selected. Thus, the four courses are:
PT59_Game_#3_#13_diagram 2.png
Accordingly, answer choice (C) is correct.
You do not have the required permissions to view the files attached to this post.

Get the most out of your LSAT Prep Plus subscription.

Analyze and track your performance with our Testing and Analytics Package.